Show a Continuous Local Martingale is a Martingale












3












$begingroup$


Let $B = (B_t)_{t≥0}$ be a standard Brownian motion started at zero, let $X=(X_t)_{t≥0}$ be a nonnegative stochastic process solving
$$dX_t = 3 , dt + 2sqrt{X_t} , dB_t qquad(X_0 = 0)$$
and let $$F(t, x) = e^{−t}x, ,,,,,,,,t geq 0,,,, x in R_+$$




  1. Need to apply Ito’s formula to $F(t, X_t)$ for $t ≥ 0$ and determine a continuous
    local martingale $(M_t)_{t≥0}$ starting at $0$ and a continuous bounded
    variation process $(A_t)_{t≥0}$ such that $F(t, X_t) = M_t+A_t$
    for $t ≥ 0$.


  2. Then Show that $(M_t)_{t≥0}$ is a martingale and compute $langle M, Mrangle$
    for $t ≥ 0$.


  3. Compute $E(int_0^tau(1/sqrt{X_t}) , dt$ when $tau = inf(t ≥ 0 : X_t = 2)$.




So far I have calculated the continuous local martingale as $$M_t = int_0^te^{-s}dX_s = 3int_0^t+2int_o^tsqrt{X_s}dB_s$$
I am unsure as how to show this is martingale and to compute $langle M, Mrangle$. And any hints for 3. would be appreciated too.










share|cite|improve this question











$endgroup$








  • 2




    $begingroup$
    how did you even derive this expression for $M_t$? Where did $e^t$ go? Can you show your exact derivation process?
    $endgroup$
    – Makina
    Dec 31 '18 at 20:56










  • $begingroup$
    Your process $M_t$ is not a martingale. Note that the stochastic integral $int_0^t f(s) , dY_s$ is a (local) martingale only if $(Y_t)_t$ is a (local) martingale...
    $endgroup$
    – saz
    Jan 1 at 9:01
















3












$begingroup$


Let $B = (B_t)_{t≥0}$ be a standard Brownian motion started at zero, let $X=(X_t)_{t≥0}$ be a nonnegative stochastic process solving
$$dX_t = 3 , dt + 2sqrt{X_t} , dB_t qquad(X_0 = 0)$$
and let $$F(t, x) = e^{−t}x, ,,,,,,,,t geq 0,,,, x in R_+$$




  1. Need to apply Ito’s formula to $F(t, X_t)$ for $t ≥ 0$ and determine a continuous
    local martingale $(M_t)_{t≥0}$ starting at $0$ and a continuous bounded
    variation process $(A_t)_{t≥0}$ such that $F(t, X_t) = M_t+A_t$
    for $t ≥ 0$.


  2. Then Show that $(M_t)_{t≥0}$ is a martingale and compute $langle M, Mrangle$
    for $t ≥ 0$.


  3. Compute $E(int_0^tau(1/sqrt{X_t}) , dt$ when $tau = inf(t ≥ 0 : X_t = 2)$.




So far I have calculated the continuous local martingale as $$M_t = int_0^te^{-s}dX_s = 3int_0^t+2int_o^tsqrt{X_s}dB_s$$
I am unsure as how to show this is martingale and to compute $langle M, Mrangle$. And any hints for 3. would be appreciated too.










share|cite|improve this question











$endgroup$








  • 2




    $begingroup$
    how did you even derive this expression for $M_t$? Where did $e^t$ go? Can you show your exact derivation process?
    $endgroup$
    – Makina
    Dec 31 '18 at 20:56










  • $begingroup$
    Your process $M_t$ is not a martingale. Note that the stochastic integral $int_0^t f(s) , dY_s$ is a (local) martingale only if $(Y_t)_t$ is a (local) martingale...
    $endgroup$
    – saz
    Jan 1 at 9:01














3












3








3


1



$begingroup$


Let $B = (B_t)_{t≥0}$ be a standard Brownian motion started at zero, let $X=(X_t)_{t≥0}$ be a nonnegative stochastic process solving
$$dX_t = 3 , dt + 2sqrt{X_t} , dB_t qquad(X_0 = 0)$$
and let $$F(t, x) = e^{−t}x, ,,,,,,,,t geq 0,,,, x in R_+$$




  1. Need to apply Ito’s formula to $F(t, X_t)$ for $t ≥ 0$ and determine a continuous
    local martingale $(M_t)_{t≥0}$ starting at $0$ and a continuous bounded
    variation process $(A_t)_{t≥0}$ such that $F(t, X_t) = M_t+A_t$
    for $t ≥ 0$.


  2. Then Show that $(M_t)_{t≥0}$ is a martingale and compute $langle M, Mrangle$
    for $t ≥ 0$.


  3. Compute $E(int_0^tau(1/sqrt{X_t}) , dt$ when $tau = inf(t ≥ 0 : X_t = 2)$.




So far I have calculated the continuous local martingale as $$M_t = int_0^te^{-s}dX_s = 3int_0^t+2int_o^tsqrt{X_s}dB_s$$
I am unsure as how to show this is martingale and to compute $langle M, Mrangle$. And any hints for 3. would be appreciated too.










share|cite|improve this question











$endgroup$




Let $B = (B_t)_{t≥0}$ be a standard Brownian motion started at zero, let $X=(X_t)_{t≥0}$ be a nonnegative stochastic process solving
$$dX_t = 3 , dt + 2sqrt{X_t} , dB_t qquad(X_0 = 0)$$
and let $$F(t, x) = e^{−t}x, ,,,,,,,,t geq 0,,,, x in R_+$$




  1. Need to apply Ito’s formula to $F(t, X_t)$ for $t ≥ 0$ and determine a continuous
    local martingale $(M_t)_{t≥0}$ starting at $0$ and a continuous bounded
    variation process $(A_t)_{t≥0}$ such that $F(t, X_t) = M_t+A_t$
    for $t ≥ 0$.


  2. Then Show that $(M_t)_{t≥0}$ is a martingale and compute $langle M, Mrangle$
    for $t ≥ 0$.


  3. Compute $E(int_0^tau(1/sqrt{X_t}) , dt$ when $tau = inf(t ≥ 0 : X_t = 2)$.




So far I have calculated the continuous local martingale as $$M_t = int_0^te^{-s}dX_s = 3int_0^t+2int_o^tsqrt{X_s}dB_s$$
I am unsure as how to show this is martingale and to compute $langle M, Mrangle$. And any hints for 3. would be appreciated too.







stochastic-processes stochastic-calculus martingales stochastic-integrals stopping-times






share|cite|improve this question















share|cite|improve this question













share|cite|improve this question




share|cite|improve this question








edited Jan 3 at 7:48









saz

81.1k861127




81.1k861127










asked Dec 31 '18 at 16:23









ZugzwangerzZugzwangerz

223




223








  • 2




    $begingroup$
    how did you even derive this expression for $M_t$? Where did $e^t$ go? Can you show your exact derivation process?
    $endgroup$
    – Makina
    Dec 31 '18 at 20:56










  • $begingroup$
    Your process $M_t$ is not a martingale. Note that the stochastic integral $int_0^t f(s) , dY_s$ is a (local) martingale only if $(Y_t)_t$ is a (local) martingale...
    $endgroup$
    – saz
    Jan 1 at 9:01














  • 2




    $begingroup$
    how did you even derive this expression for $M_t$? Where did $e^t$ go? Can you show your exact derivation process?
    $endgroup$
    – Makina
    Dec 31 '18 at 20:56










  • $begingroup$
    Your process $M_t$ is not a martingale. Note that the stochastic integral $int_0^t f(s) , dY_s$ is a (local) martingale only if $(Y_t)_t$ is a (local) martingale...
    $endgroup$
    – saz
    Jan 1 at 9:01








2




2




$begingroup$
how did you even derive this expression for $M_t$? Where did $e^t$ go? Can you show your exact derivation process?
$endgroup$
– Makina
Dec 31 '18 at 20:56




$begingroup$
how did you even derive this expression for $M_t$? Where did $e^t$ go? Can you show your exact derivation process?
$endgroup$
– Makina
Dec 31 '18 at 20:56












$begingroup$
Your process $M_t$ is not a martingale. Note that the stochastic integral $int_0^t f(s) , dY_s$ is a (local) martingale only if $(Y_t)_t$ is a (local) martingale...
$endgroup$
– saz
Jan 1 at 9:01




$begingroup$
Your process $M_t$ is not a martingale. Note that the stochastic integral $int_0^t f(s) , dY_s$ is a (local) martingale only if $(Y_t)_t$ is a (local) martingale...
$endgroup$
– saz
Jan 1 at 9:01










1 Answer
1






active

oldest

votes


















4












$begingroup$

Part 1: Itô's formula shows



$$F(t,X_t) = int_0^t e^{-s} , dX_s - int_0^t e^{-s} X_s , ds = M_t+A_t$$



where



$$M_t = 2 int_0^t e ^{-s} sqrt{X_s} , dB_s quad text{and} quad A_t := int_0^t e^{-s} (3-X_s) , ds.$$



Part 2: Set $tau_r := inf{t geq 0; X_t geq r}$. By the very definition of $X$, we have



$$X_{t wedge tau_r} = 3 (t wedge tau_r) + 2 int_0^{t wedge tau_r} sqrt{X_s} , dB_s$$



and so



$$mathbb{E}(X_{t wedge tau_r}) = 3 mathbb{E}(t wedge tau_r) leq 3t.$$



This implies that $f(s,omega) := 2 e^{-s} sqrt{X_s(omega)}$ satisfies $mathbb{E}(int_0^t f(s)^2 , ds) < infty$ for any $t>0$, and therefore $$M_t = int_0^t f(s) , dB_s = 2 int_0^t e ^{-s} sqrt{X_s} , dB_s$$ is a martingale. It is well known that for any stochastic integral of this form, the quadratic variation $langle M rangle$ is given by $$langle M rangle_t = int_0^t f(s)^2 , ds = 4 int_0^t e^{-2s} X_s , ds.$$



Part 3: Set $g(x) := sqrt{x}$. Applying formally Itô's formula (see the remark below) we find



$$begin{align*} g(X_t) = sqrt{X_t} &= frac{1}{2} int_0^t frac{1}{sqrt{X_s}} , dX_s - frac{1}{8} int_0^t frac{1}{X_s^{3/2}} , underbrace{dlangle X rangle_s}_{=4 X_s , ds} \ &= int_0^t dB_s + int_0^t frac{1}{sqrt{X_s}} , ds. end{align*}$$



Hence,



$$mathbb{E}(sqrt{X_{t wedge tau}}) = underbrace{mathbb{E}(B_{t wedge tau})}_{=0} + mathbb{E} left( int_0^{t wedge tau} frac{1}{sqrt{X_s}} , ds right). tag{1}$$



Since $0 leq X_{t wedge tau} leq 2$ this implies



$$frac{1}{sqrt{2}} mathbb{E}(t wedge tau) leq mathbb{E}(X_{t wedge tau}) leq sqrt{2},$$



and by the monotone convergence theorem this gives $mathbb{E}(tau)<infty$; in particular, $tau<infty$ almost surely. Applying the dominated convergence theorem and the monotone convergence theorem, we conclude from $(1)$ that



$$mathbb{E} left( int_0^{tau} frac{1}{sqrt{X_s}} , ds right) = mathbb{E}(sqrt{X_{tau}}) = sqrt{2}.$$



Remark: Since $g(x) = sqrt{x}$ is not twice continuously differentiable, we cannot apply the classical version of Itô's formula. To make the above calculations rigorous, one has to use a truncation technique. To this end choose a smooth function $chi$ such that $chi(x)=0$ for $|x|<1/2$ and $chi(x)=1$ for $|x| geq 1$, and define $$chi_n(x) := chi left( n x right), qquad x in mathbb{R}.$$
Since $x mapsto g_n(x) := g(x) chi_n(x)$ is twice continuously differentiable, we may apply Itô's formula and then we can use $g_n(x) to g(x)$ to obtain the above result.






share|cite|improve this answer











$endgroup$













  • $begingroup$
    in part 2 you claim that the expectation of the stochastic integral is zero. Are you able to do so because of stopping time (which makes the integrand bounded i.e it can't explode to infinity)
    $endgroup$
    – Makina
    Jan 4 at 18:14












  • $begingroup$
    @Makina Yes, exactly.
    $endgroup$
    – saz
    Jan 4 at 18:24











Your Answer





StackExchange.ifUsing("editor", function () {
return StackExchange.using("mathjaxEditing", function () {
StackExchange.MarkdownEditor.creationCallbacks.add(function (editor, postfix) {
StackExchange.mathjaxEditing.prepareWmdForMathJax(editor, postfix, [["$", "$"], ["\\(","\\)"]]);
});
});
}, "mathjax-editing");

StackExchange.ready(function() {
var channelOptions = {
tags: "".split(" "),
id: "69"
};
initTagRenderer("".split(" "), "".split(" "), channelOptions);

StackExchange.using("externalEditor", function() {
// Have to fire editor after snippets, if snippets enabled
if (StackExchange.settings.snippets.snippetsEnabled) {
StackExchange.using("snippets", function() {
createEditor();
});
}
else {
createEditor();
}
});

function createEditor() {
StackExchange.prepareEditor({
heartbeatType: 'answer',
autoActivateHeartbeat: false,
convertImagesToLinks: true,
noModals: true,
showLowRepImageUploadWarning: true,
reputationToPostImages: 10,
bindNavPrevention: true,
postfix: "",
imageUploader: {
brandingHtml: "Powered by u003ca class="icon-imgur-white" href="https://imgur.com/"u003eu003c/au003e",
contentPolicyHtml: "User contributions licensed under u003ca href="https://creativecommons.org/licenses/by-sa/3.0/"u003ecc by-sa 3.0 with attribution requiredu003c/au003e u003ca href="https://stackoverflow.com/legal/content-policy"u003e(content policy)u003c/au003e",
allowUrls: true
},
noCode: true, onDemand: true,
discardSelector: ".discard-answer"
,immediatelyShowMarkdownHelp:true
});


}
});














draft saved

draft discarded


















StackExchange.ready(
function () {
StackExchange.openid.initPostLogin('.new-post-login', 'https%3a%2f%2fmath.stackexchange.com%2fquestions%2f3057849%2fshow-a-continuous-local-martingale-is-a-martingale%23new-answer', 'question_page');
}
);

Post as a guest















Required, but never shown

























1 Answer
1






active

oldest

votes








1 Answer
1






active

oldest

votes









active

oldest

votes






active

oldest

votes









4












$begingroup$

Part 1: Itô's formula shows



$$F(t,X_t) = int_0^t e^{-s} , dX_s - int_0^t e^{-s} X_s , ds = M_t+A_t$$



where



$$M_t = 2 int_0^t e ^{-s} sqrt{X_s} , dB_s quad text{and} quad A_t := int_0^t e^{-s} (3-X_s) , ds.$$



Part 2: Set $tau_r := inf{t geq 0; X_t geq r}$. By the very definition of $X$, we have



$$X_{t wedge tau_r} = 3 (t wedge tau_r) + 2 int_0^{t wedge tau_r} sqrt{X_s} , dB_s$$



and so



$$mathbb{E}(X_{t wedge tau_r}) = 3 mathbb{E}(t wedge tau_r) leq 3t.$$



This implies that $f(s,omega) := 2 e^{-s} sqrt{X_s(omega)}$ satisfies $mathbb{E}(int_0^t f(s)^2 , ds) < infty$ for any $t>0$, and therefore $$M_t = int_0^t f(s) , dB_s = 2 int_0^t e ^{-s} sqrt{X_s} , dB_s$$ is a martingale. It is well known that for any stochastic integral of this form, the quadratic variation $langle M rangle$ is given by $$langle M rangle_t = int_0^t f(s)^2 , ds = 4 int_0^t e^{-2s} X_s , ds.$$



Part 3: Set $g(x) := sqrt{x}$. Applying formally Itô's formula (see the remark below) we find



$$begin{align*} g(X_t) = sqrt{X_t} &= frac{1}{2} int_0^t frac{1}{sqrt{X_s}} , dX_s - frac{1}{8} int_0^t frac{1}{X_s^{3/2}} , underbrace{dlangle X rangle_s}_{=4 X_s , ds} \ &= int_0^t dB_s + int_0^t frac{1}{sqrt{X_s}} , ds. end{align*}$$



Hence,



$$mathbb{E}(sqrt{X_{t wedge tau}}) = underbrace{mathbb{E}(B_{t wedge tau})}_{=0} + mathbb{E} left( int_0^{t wedge tau} frac{1}{sqrt{X_s}} , ds right). tag{1}$$



Since $0 leq X_{t wedge tau} leq 2$ this implies



$$frac{1}{sqrt{2}} mathbb{E}(t wedge tau) leq mathbb{E}(X_{t wedge tau}) leq sqrt{2},$$



and by the monotone convergence theorem this gives $mathbb{E}(tau)<infty$; in particular, $tau<infty$ almost surely. Applying the dominated convergence theorem and the monotone convergence theorem, we conclude from $(1)$ that



$$mathbb{E} left( int_0^{tau} frac{1}{sqrt{X_s}} , ds right) = mathbb{E}(sqrt{X_{tau}}) = sqrt{2}.$$



Remark: Since $g(x) = sqrt{x}$ is not twice continuously differentiable, we cannot apply the classical version of Itô's formula. To make the above calculations rigorous, one has to use a truncation technique. To this end choose a smooth function $chi$ such that $chi(x)=0$ for $|x|<1/2$ and $chi(x)=1$ for $|x| geq 1$, and define $$chi_n(x) := chi left( n x right), qquad x in mathbb{R}.$$
Since $x mapsto g_n(x) := g(x) chi_n(x)$ is twice continuously differentiable, we may apply Itô's formula and then we can use $g_n(x) to g(x)$ to obtain the above result.






share|cite|improve this answer











$endgroup$













  • $begingroup$
    in part 2 you claim that the expectation of the stochastic integral is zero. Are you able to do so because of stopping time (which makes the integrand bounded i.e it can't explode to infinity)
    $endgroup$
    – Makina
    Jan 4 at 18:14












  • $begingroup$
    @Makina Yes, exactly.
    $endgroup$
    – saz
    Jan 4 at 18:24
















4












$begingroup$

Part 1: Itô's formula shows



$$F(t,X_t) = int_0^t e^{-s} , dX_s - int_0^t e^{-s} X_s , ds = M_t+A_t$$



where



$$M_t = 2 int_0^t e ^{-s} sqrt{X_s} , dB_s quad text{and} quad A_t := int_0^t e^{-s} (3-X_s) , ds.$$



Part 2: Set $tau_r := inf{t geq 0; X_t geq r}$. By the very definition of $X$, we have



$$X_{t wedge tau_r} = 3 (t wedge tau_r) + 2 int_0^{t wedge tau_r} sqrt{X_s} , dB_s$$



and so



$$mathbb{E}(X_{t wedge tau_r}) = 3 mathbb{E}(t wedge tau_r) leq 3t.$$



This implies that $f(s,omega) := 2 e^{-s} sqrt{X_s(omega)}$ satisfies $mathbb{E}(int_0^t f(s)^2 , ds) < infty$ for any $t>0$, and therefore $$M_t = int_0^t f(s) , dB_s = 2 int_0^t e ^{-s} sqrt{X_s} , dB_s$$ is a martingale. It is well known that for any stochastic integral of this form, the quadratic variation $langle M rangle$ is given by $$langle M rangle_t = int_0^t f(s)^2 , ds = 4 int_0^t e^{-2s} X_s , ds.$$



Part 3: Set $g(x) := sqrt{x}$. Applying formally Itô's formula (see the remark below) we find



$$begin{align*} g(X_t) = sqrt{X_t} &= frac{1}{2} int_0^t frac{1}{sqrt{X_s}} , dX_s - frac{1}{8} int_0^t frac{1}{X_s^{3/2}} , underbrace{dlangle X rangle_s}_{=4 X_s , ds} \ &= int_0^t dB_s + int_0^t frac{1}{sqrt{X_s}} , ds. end{align*}$$



Hence,



$$mathbb{E}(sqrt{X_{t wedge tau}}) = underbrace{mathbb{E}(B_{t wedge tau})}_{=0} + mathbb{E} left( int_0^{t wedge tau} frac{1}{sqrt{X_s}} , ds right). tag{1}$$



Since $0 leq X_{t wedge tau} leq 2$ this implies



$$frac{1}{sqrt{2}} mathbb{E}(t wedge tau) leq mathbb{E}(X_{t wedge tau}) leq sqrt{2},$$



and by the monotone convergence theorem this gives $mathbb{E}(tau)<infty$; in particular, $tau<infty$ almost surely. Applying the dominated convergence theorem and the monotone convergence theorem, we conclude from $(1)$ that



$$mathbb{E} left( int_0^{tau} frac{1}{sqrt{X_s}} , ds right) = mathbb{E}(sqrt{X_{tau}}) = sqrt{2}.$$



Remark: Since $g(x) = sqrt{x}$ is not twice continuously differentiable, we cannot apply the classical version of Itô's formula. To make the above calculations rigorous, one has to use a truncation technique. To this end choose a smooth function $chi$ such that $chi(x)=0$ for $|x|<1/2$ and $chi(x)=1$ for $|x| geq 1$, and define $$chi_n(x) := chi left( n x right), qquad x in mathbb{R}.$$
Since $x mapsto g_n(x) := g(x) chi_n(x)$ is twice continuously differentiable, we may apply Itô's formula and then we can use $g_n(x) to g(x)$ to obtain the above result.






share|cite|improve this answer











$endgroup$













  • $begingroup$
    in part 2 you claim that the expectation of the stochastic integral is zero. Are you able to do so because of stopping time (which makes the integrand bounded i.e it can't explode to infinity)
    $endgroup$
    – Makina
    Jan 4 at 18:14












  • $begingroup$
    @Makina Yes, exactly.
    $endgroup$
    – saz
    Jan 4 at 18:24














4












4








4





$begingroup$

Part 1: Itô's formula shows



$$F(t,X_t) = int_0^t e^{-s} , dX_s - int_0^t e^{-s} X_s , ds = M_t+A_t$$



where



$$M_t = 2 int_0^t e ^{-s} sqrt{X_s} , dB_s quad text{and} quad A_t := int_0^t e^{-s} (3-X_s) , ds.$$



Part 2: Set $tau_r := inf{t geq 0; X_t geq r}$. By the very definition of $X$, we have



$$X_{t wedge tau_r} = 3 (t wedge tau_r) + 2 int_0^{t wedge tau_r} sqrt{X_s} , dB_s$$



and so



$$mathbb{E}(X_{t wedge tau_r}) = 3 mathbb{E}(t wedge tau_r) leq 3t.$$



This implies that $f(s,omega) := 2 e^{-s} sqrt{X_s(omega)}$ satisfies $mathbb{E}(int_0^t f(s)^2 , ds) < infty$ for any $t>0$, and therefore $$M_t = int_0^t f(s) , dB_s = 2 int_0^t e ^{-s} sqrt{X_s} , dB_s$$ is a martingale. It is well known that for any stochastic integral of this form, the quadratic variation $langle M rangle$ is given by $$langle M rangle_t = int_0^t f(s)^2 , ds = 4 int_0^t e^{-2s} X_s , ds.$$



Part 3: Set $g(x) := sqrt{x}$. Applying formally Itô's formula (see the remark below) we find



$$begin{align*} g(X_t) = sqrt{X_t} &= frac{1}{2} int_0^t frac{1}{sqrt{X_s}} , dX_s - frac{1}{8} int_0^t frac{1}{X_s^{3/2}} , underbrace{dlangle X rangle_s}_{=4 X_s , ds} \ &= int_0^t dB_s + int_0^t frac{1}{sqrt{X_s}} , ds. end{align*}$$



Hence,



$$mathbb{E}(sqrt{X_{t wedge tau}}) = underbrace{mathbb{E}(B_{t wedge tau})}_{=0} + mathbb{E} left( int_0^{t wedge tau} frac{1}{sqrt{X_s}} , ds right). tag{1}$$



Since $0 leq X_{t wedge tau} leq 2$ this implies



$$frac{1}{sqrt{2}} mathbb{E}(t wedge tau) leq mathbb{E}(X_{t wedge tau}) leq sqrt{2},$$



and by the monotone convergence theorem this gives $mathbb{E}(tau)<infty$; in particular, $tau<infty$ almost surely. Applying the dominated convergence theorem and the monotone convergence theorem, we conclude from $(1)$ that



$$mathbb{E} left( int_0^{tau} frac{1}{sqrt{X_s}} , ds right) = mathbb{E}(sqrt{X_{tau}}) = sqrt{2}.$$



Remark: Since $g(x) = sqrt{x}$ is not twice continuously differentiable, we cannot apply the classical version of Itô's formula. To make the above calculations rigorous, one has to use a truncation technique. To this end choose a smooth function $chi$ such that $chi(x)=0$ for $|x|<1/2$ and $chi(x)=1$ for $|x| geq 1$, and define $$chi_n(x) := chi left( n x right), qquad x in mathbb{R}.$$
Since $x mapsto g_n(x) := g(x) chi_n(x)$ is twice continuously differentiable, we may apply Itô's formula and then we can use $g_n(x) to g(x)$ to obtain the above result.






share|cite|improve this answer











$endgroup$



Part 1: Itô's formula shows



$$F(t,X_t) = int_0^t e^{-s} , dX_s - int_0^t e^{-s} X_s , ds = M_t+A_t$$



where



$$M_t = 2 int_0^t e ^{-s} sqrt{X_s} , dB_s quad text{and} quad A_t := int_0^t e^{-s} (3-X_s) , ds.$$



Part 2: Set $tau_r := inf{t geq 0; X_t geq r}$. By the very definition of $X$, we have



$$X_{t wedge tau_r} = 3 (t wedge tau_r) + 2 int_0^{t wedge tau_r} sqrt{X_s} , dB_s$$



and so



$$mathbb{E}(X_{t wedge tau_r}) = 3 mathbb{E}(t wedge tau_r) leq 3t.$$



This implies that $f(s,omega) := 2 e^{-s} sqrt{X_s(omega)}$ satisfies $mathbb{E}(int_0^t f(s)^2 , ds) < infty$ for any $t>0$, and therefore $$M_t = int_0^t f(s) , dB_s = 2 int_0^t e ^{-s} sqrt{X_s} , dB_s$$ is a martingale. It is well known that for any stochastic integral of this form, the quadratic variation $langle M rangle$ is given by $$langle M rangle_t = int_0^t f(s)^2 , ds = 4 int_0^t e^{-2s} X_s , ds.$$



Part 3: Set $g(x) := sqrt{x}$. Applying formally Itô's formula (see the remark below) we find



$$begin{align*} g(X_t) = sqrt{X_t} &= frac{1}{2} int_0^t frac{1}{sqrt{X_s}} , dX_s - frac{1}{8} int_0^t frac{1}{X_s^{3/2}} , underbrace{dlangle X rangle_s}_{=4 X_s , ds} \ &= int_0^t dB_s + int_0^t frac{1}{sqrt{X_s}} , ds. end{align*}$$



Hence,



$$mathbb{E}(sqrt{X_{t wedge tau}}) = underbrace{mathbb{E}(B_{t wedge tau})}_{=0} + mathbb{E} left( int_0^{t wedge tau} frac{1}{sqrt{X_s}} , ds right). tag{1}$$



Since $0 leq X_{t wedge tau} leq 2$ this implies



$$frac{1}{sqrt{2}} mathbb{E}(t wedge tau) leq mathbb{E}(X_{t wedge tau}) leq sqrt{2},$$



and by the monotone convergence theorem this gives $mathbb{E}(tau)<infty$; in particular, $tau<infty$ almost surely. Applying the dominated convergence theorem and the monotone convergence theorem, we conclude from $(1)$ that



$$mathbb{E} left( int_0^{tau} frac{1}{sqrt{X_s}} , ds right) = mathbb{E}(sqrt{X_{tau}}) = sqrt{2}.$$



Remark: Since $g(x) = sqrt{x}$ is not twice continuously differentiable, we cannot apply the classical version of Itô's formula. To make the above calculations rigorous, one has to use a truncation technique. To this end choose a smooth function $chi$ such that $chi(x)=0$ for $|x|<1/2$ and $chi(x)=1$ for $|x| geq 1$, and define $$chi_n(x) := chi left( n x right), qquad x in mathbb{R}.$$
Since $x mapsto g_n(x) := g(x) chi_n(x)$ is twice continuously differentiable, we may apply Itô's formula and then we can use $g_n(x) to g(x)$ to obtain the above result.







share|cite|improve this answer














share|cite|improve this answer



share|cite|improve this answer








edited Jan 3 at 15:03

























answered Jan 2 at 20:12









sazsaz

81.1k861127




81.1k861127












  • $begingroup$
    in part 2 you claim that the expectation of the stochastic integral is zero. Are you able to do so because of stopping time (which makes the integrand bounded i.e it can't explode to infinity)
    $endgroup$
    – Makina
    Jan 4 at 18:14












  • $begingroup$
    @Makina Yes, exactly.
    $endgroup$
    – saz
    Jan 4 at 18:24


















  • $begingroup$
    in part 2 you claim that the expectation of the stochastic integral is zero. Are you able to do so because of stopping time (which makes the integrand bounded i.e it can't explode to infinity)
    $endgroup$
    – Makina
    Jan 4 at 18:14












  • $begingroup$
    @Makina Yes, exactly.
    $endgroup$
    – saz
    Jan 4 at 18:24
















$begingroup$
in part 2 you claim that the expectation of the stochastic integral is zero. Are you able to do so because of stopping time (which makes the integrand bounded i.e it can't explode to infinity)
$endgroup$
– Makina
Jan 4 at 18:14






$begingroup$
in part 2 you claim that the expectation of the stochastic integral is zero. Are you able to do so because of stopping time (which makes the integrand bounded i.e it can't explode to infinity)
$endgroup$
– Makina
Jan 4 at 18:14














$begingroup$
@Makina Yes, exactly.
$endgroup$
– saz
Jan 4 at 18:24




$begingroup$
@Makina Yes, exactly.
$endgroup$
– saz
Jan 4 at 18:24


















draft saved

draft discarded




















































Thanks for contributing an answer to Mathematics Stack Exchange!


  • Please be sure to answer the question. Provide details and share your research!

But avoid



  • Asking for help, clarification, or responding to other answers.

  • Making statements based on opinion; back them up with references or personal experience.


Use MathJax to format equations. MathJax reference.


To learn more, see our tips on writing great answers.




draft saved


draft discarded














StackExchange.ready(
function () {
StackExchange.openid.initPostLogin('.new-post-login', 'https%3a%2f%2fmath.stackexchange.com%2fquestions%2f3057849%2fshow-a-continuous-local-martingale-is-a-martingale%23new-answer', 'question_page');
}
);

Post as a guest















Required, but never shown





















































Required, but never shown














Required, but never shown












Required, but never shown







Required, but never shown

































Required, but never shown














Required, but never shown












Required, but never shown







Required, but never shown







Popular posts from this blog

Quarter-circle Tiles

build a pushdown automaton that recognizes the reverse language of a given pushdown automaton?

Mont Emei